diff --git "a/IMO/md/en-IMO-2018-notes.md" "b/IMO/md/en-IMO-2018-notes.md" new file mode 100644--- /dev/null +++ "b/IMO/md/en-IMO-2018-notes.md" @@ -0,0 +1,413 @@ + +# IMO 2018 Solution Notes + +Evan Chen 《陳誼廷》 + +18 October 2025 + +This is a compilation of solutions for the 2018 IMO. The ideas of the solution are a mix of my own work, the solutions provided by the competition organizers, and solutions found by the community. However, all the writing is maintained by me. + +These notes will tend to be a bit more advanced and terse than the "official" solutions from the organizers. In particular, if a theorem or technique is not known to beginners but is still considered "standard", then I often prefer to use this theory anyways, rather than try to work around or conceal it. For example, in geometry problems I typically use directed angles without further comment, rather than awkwardly work around configuration issues. Similarly, sentences like "let \(\mathbb{R}\) denote the set of real numbers" are typically omitted entirely. + +Corrections and comments are welcome! + +## Contents + +0 Problems 2 + +1 Solutions to Day 1 3 + +1.1 IMO 2018/1, proposed by Silouanos Brazitikos, Vangelis Psyxas, Michael + +Sarantis (HEL) 3 + +1.2 IMO 2018/2, proposed by Patrik Bak (SVK) 6 + +1.3 IMO 2018/3, proposed by Morteza Saghafian (IRN) 8 + +2 Solutions to Day 2 10 + +2.1 IMO 2018/4, proposed by Gurgen Asatryan (ARM) 10 + +2.2 IMO 2018/5, proposed by Bayarmagnai Gombodorj (MNG) 11 + +2.3 IMO 2018/6, proposed by Tomasz Ciesla (POL) 13 + + + +## Problems + +1. Let \(\Gamma\) be the circumcircle of acute triangle \(A B C\) . Points \(D\) and \(E\) lie on segments \(A B\) and \(A C\) , respectively, such that \(A D = A E\) . The perpendicular bisectors of \(\overline{{B D}}\) and \(\overline{{C E}}\) intersect the minor arcs \(A B\) and \(A C\) of \(\Gamma\) at points \(F\) and \(G\) , respectively. Prove that the lines \(D E\) and \(F G\) are parallel. + +2. Find all integers \(n\geq 3\) for which there exist real numbers \(a_{1},a_{2},\ldots ,a_{n}\) satisfying + +\[a_{i}a_{i + 1} + 1 = a_{i + 2}\] + +for \(i = 1,2,\ldots ,n\) , where indices are taken modulo \(n\) + +3. An anti-Pascal triangle is an equilateral triangular array of numbers such that, except for the numbers in the bottom row, each number is the absolute value of the difference of the two numbers immediately below it. For example, the following array is an anti-Pascal triangle with four rows which contains every integer from 1 to 10. + +\[4\] \[2 6\] \[5 7 1\] \[8 3 10 9\] + +Does there exist an anti- Pascal triangle with 2018 rows which contains every integer from 1 to \(1 + 2 + \dots +2018\) ? + +4. A site is any point \((x,y)\) in the plane for which \(x,y\in \{1,\ldots ,20\}\) . Initially, each of the 400 sites is unoccupied. Any and Ben take turns placing stones on unoccupied sites, with Amy going first; Amy has the additional restriction that no two of her stones may be at a distance equal to \(\sqrt{5}\) . They stop once either player cannot move. Find the greatest \(K\) such that Amy can ensure that she places at least \(K\) stones. + +5. Let \(a_{1}\) , \(a_{2}\) , ... be an infinite sequence of positive integers, and \(N\) a positive integer. Suppose that for all integers \(n\geq N\) , the expression + +\[\frac{a_{1}}{a_{2}} +\frac{a_{2}}{a_{3}} +\dots +\frac{a_{n - 1}}{a_{n}} +\frac{a_{n}}{a_{1}}\] + +is an integer. Prove that \((a_{n})\) is eventually constant. + +6. A convex quadrilateral \(A B C D\) satisfies \(A B\cdot C D = B C\cdot D A\) . Point \(X\) lies inside \(A B C D\) so that + +\[\angle X A B = \angle X C D\quad \mathrm{and}\quad \angle X B C = \angle X D A.\] + +Prove that \(\angle B X A + \angle D X C = 180^{\circ}\) + + + +## \(\S 1\) Solutions to Day 1 + +## \(\S 1.1\) IMO 2018/1, proposed by Silouanos Brazitikos, Vangelis Psyxas, Michael Sarantis (HEL) + +Available online at https://aops.com/community/p10626500. + +## Problem statement + +Let \(\Gamma\) be the circumcircle of acute triangle \(ABC\) . Points \(D\) and \(E\) lie on segments \(AB\) and \(AC\) , respectively, such that \(AD = AE\) . The perpendicular bisectors of \(\overline{BD}\) and \(\overline{CE}\) intersect the minor arcs \(AB\) and \(AC\) of \(\Gamma\) at points \(F\) and \(G\) , respectively. Prove that the lines \(DE\) and \(FG\) are parallel. + +We present a synthetic solution from the IMO shortlist as well as a complex numbers approach. We also outline a trig solution (the one I found at IMO), and a fourth solution from Derek Liu. + +\(\P\) Synthetic solution (from Shortlist). Construct parallelograms \(AXFD\) and \(AEGY\) , noting that \(X\) and \(Y\) lie on \(\Gamma\) . As \(\overline{XF} \parallel \overline{AB}\) we can let \(M\) denote the midpoint of minor arcs \(\overline{XF}\) and \(\overline{AB}\) (which coincide). Define \(N\) similarly. + +![](data:image/jpeg;base64,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) + + +Observe that \(XF = AD = AE = YG\) , so arcs \(\widehat{XF}\) and \(\widehat{YG}\) have equal measure; hence arcs \(\widehat{MF}\) and \(\widehat{NG}\) have equal measure; therefore \(\widehat{MN} \parallel \widehat{FG}\) . + +Since \(\widehat{MN}\) and \(\widehat{DE}\) are both perpendicular to the \(\angle A\) bisector, we're done. + +\(\P\) Complex numbers solution. Let \(b\) , \(c\) , \(f\) , \(g\) , \(a\) be as usual. Note that + +\[d - a = \left(2\cdot \frac{f + a + b - ab\overline{f}}{2} -b\right) - a = f - \frac{ab}{f}\] \[e - a = g - \frac{ac}{g}\] + + + +We are given \(AD = AE\) from which one deduces + +\[\left(\frac{e - a}{d - a}\right)^{2} = \frac{c}{b}\Longrightarrow \frac{(g^{2} - a c)^{2}}{(f^{2} - a b)^{2}} = \frac{g^{2}c}{f^{2}b}\] \[\Longrightarrow b c(b g^{2} - c f^{2})a^{2} = g^{2}f^{4}c - f^{2}g^{4}b = f^{2}g^{2}(f^{2}c - g^{2}b)\] \[\Longrightarrow b c\cdot a^{2} = (f g)^{2}\Longrightarrow \left(-\frac{f g}{a}\right)^{2} = b c.\] + +Since \(\frac{- f g}{a}\) is the point \(X\) on the circle with \(\overline{{A X}}\perp \overline{{F G}}\) , we conclude \(\overline{{F G}}\) is either parallel or perpendicular to the \(\angle A\) - bisector; it must the latter since the \(\angle A\) - bisector separates the two minor arcs. + +\(\P\) Trig solution (outline). Let \(\ell\) denote the \(\angle A\) bisector. Fix \(D\) and \(F\) . We define the phantom point \(G^{\prime}\) such that \(\overline{{F G^{\prime}}}\perp \ell\) and \(E^{\prime}\) on side \(\overline{{A C}}\) such that \(G E^{\prime} = G C\) + +Claim (Converse of the IMO problem) — We have \(AD = AE^{\prime}\) , so that \(E = E^{\prime}\) . + +Proof. Since \(\overline{{F G^{\prime}}}\perp \ell\) , one can deduce \(\angle F B D = \frac{1}{2} C + x\) and \(\angle G C A = \frac{1}{2} B + x\) for some \(x\) . (One fast way to see this is to note that \(\overline{{F G}}\parallel \overline{{M N}}\) where \(M\) and \(N\) are in the first solution.) Then \(\angle F A B = \frac{1}{2} C - x\) and \(\angle G A C = \frac{1}{2} B - x\) . + +Let \(R\) be the circumradius. Now, by the law of sines, + +\[B F = 2R\sin \left(\frac{1}{2} C - x\right).\] + +From there we get + +\[B D = 2\cdot B F\cos \left(\frac{1}{2} C + x\right) = 4R\cos \left(\frac{1}{2} C + x\right)\sin \left(\frac{1}{2} C - x\right)\] \[D A = A B - B D = 2R\sin C - 4R\cos \left(\frac{1}{2} C + x\right)\sin \left(\frac{1}{2} C - x\right)\] \[\qquad = 2R\left[\sin C - 2\cos \left(\frac{1}{2} C + x\right)\sin \left(\frac{1}{2} C - x\right)\right]\] \[\qquad = 2R\left[\sin C - (\sin C - \sin 2x)\right] = 2R\sin 2x.\] + +A similar calculation gives \(A E^{\prime} = 2R\sin 2x\) as needed. + +Thus, \(\overline{{F G^{\prime}}}\parallel \overline{{D E}}\) , so \(G = G^{\prime}\) as well. This concludes the proof. + +\(\P\) Synthetic solution from Derek Liu. Let lines \(F D\) and \(G E\) intersect \(\Gamma\) again at \(J\) and \(K\) , respectively. + + +![](data:image/jpeg;base64,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) + + +Notice that \(\triangle BFD \sim \triangle JAD\) ; as \(FB = FD\) , it follows that \(AJ = AD\) . Likewise, \(\triangle CGE \sim \triangle KAE\) and \(GC = GE\) , so \(AK = AE\) . Hence, + +\[AK = AE = AD = AJ,\] + +so \(DEJK\) is cyclic with center \(A\) . + +It follows that + +\[\angle KED = \angle KJD = \angle KJF = \angle KGF,\] + +so we're done. + +Remark. Note that \(K\) and \(J\) must be distinct for this solution to work. Since \(G\) and \(K\) lie on opposite sides of \(AC\) , \(K\) is on major arc \(ABC\) . As \(AK = AD = AE \leq \min (AB, AC)\) , \(K\) lies on minor arc \(AB\) . Similarly, \(J\) lies on minor arc \(AC\) , so \(K \neq J\) . + + + +## \(\S 1.2\) IMO 2018/2, proposed by Patrik Bak (SVK) + +Available online at https://aops.com/community/p10626524. + +## Problem statement + +Find all integers \(n \geq 3\) for which there exist real numbers \(a_1, a_2, \ldots , a_n\) satisfying + +\[a_{i}a_{i + 1} + 1 = a_{i + 2}\] + +for \(i = 1,2,\ldots ,n\) , where indices are taken modulo \(n\) + +The answer is \(3 \mid n\) , achieved by \((- 1, - 1,2, - 1, - 1,2,\ldots)\) . We present two solutions. + +\(\P\) First solution by inequalities. We compute \(a_{i}a_{i + 1}a_{i + 2}\) in two ways: + +\[a_{i}a_{i + 1}a_{i + 2} = [a_{i + 2} - 1]a_{i + 2} = a_{i + 2}^{2} - a_{i + 2}\] \[\qquad = a_{i}[a_{i + 3} - 1] = a_{i}a_{i + 3} - a_{i}.\] + +Cyclically summing \(a_{i + 2}^{2} - a_{i + 2} = a_{i}a_{i + 3} - a_{i}\) then gives + +\[\sum_{i}a_{i + 2}^{2} = \sum_{i}a_{i}a_{i + 3}\iff \sum_{\mathrm{cyc}}(a_{i} - a_{i + 3})^{2} = 0.\] + +This means for inequality reasons the sequence is 3- periodic. Since the sequence is clearly not 1- periodic, as \(x^{2} + 1 = x\) has no real solutions. Thus \(3 \mid n\) . + +\(\P\) Second solution by sign counting. Extend \(a_{n}\) to be a periodic sequence. The idea is to look at the signs, and show the sequence of the signs must be \(- - +\) repeated. This takes several steps: + +- The pattern \(- - -\) is impossible. Obvious, since the third term should be \(>1\) . + +- The pattern \(+ +\) is impossible. Then the sequence becomes strictly increasing, hence may not be periodic. + +- Zeros are impossible. If \(a_{1} = 0\) , then \(a_{2} = 0\) , \(a_{3} > 0\) , \(a_{4} > 0\) , which gives the impossible \(+ +\) . + +- The pattern \(- - + - +\) is impossible. Compute the terms: + +\[a_{1} = -x< 0\] \[a_{2} = -y< 0\] \[a_{3} = 1 + xy > 1\] \[a_{4} = 1 - y(1 + xy)< 0\] \[a_{5} = 1 + (1 + xy)(1 - y(1 + xy))< 1.\] + +But now + +\[a_{6} - a_{5} = (1 + a_{5}a_{4}) - (1 + a_{3}a_{4}) = a_{4}(a_{5} - a_{3}) > 0\] + +since \(a_{5} > 1 > a_{3}\) . This means we have the impossible \(+ +\) pattern. + + + +- The infinite alternating pattern \(- + - + - + - + \ldots\) is impossible. Note that + +\[a_{1}a_{2} + 1 = a_{3}< 0< a_{4} = 1 + a_{2}a_{3}\Rightarrow a_{1}< a_{3}\] + +since \(a_{2} > 0\) ; extending this we get \(a_{1}< a_{3}< a_{5}< \ldots\) which contradicts the periodicity. + +We finally collate the logic of sign patterns. Since the pattern is not alternating, there must be - - somewhere. Afterwards must be \(^+\) , and then after that must be two minus signs (since one minus sign is impossible by impossibility of \(- - + - +\) and \(- - -\) is also forbidden); thus we get the periodic \(- - +\) as desired. + + + +## \(\S 1.3\) IMO 2018/3, proposed by Morteza Saghafian (IRN) + +Available online at https://aops.com/community/p10626557. + +## Problem statement + +An anti- Pascal triangle is an equilateral triangular array of numbers such that, except for the numbers in the bottom row, each number is the absolute value of the difference of the two numbers immediately below it. For example, the following array is an anti- Pascal triangle with four rows which contains every integer from 1 to 10. + +Does there exist an anti- Pascal triangle with 2018 rows which contains every integer from 1 to \(1 + 2 + \dots +2018?\) + +The answer is no, there is no anti- Pascal triangle with the required properties. + +Let \(n = 2018\) and \(N = 1 + 2 + \dots +n\) . For every number \(d\) not in the bottom row, draw an arrow from \(d\) to the larger of the two numbers below it (i.e. if \(d = a - b\) , draw \(d\rightarrow a\) ). This creates an oriented forest (which looks like lightning strikes). + +Consider the directed path starting from the top vertex \(A\) . Starting from the first number, it increments by at least \(1 + 2 + \dots +n\) , since the increments at each step in the path are distinct; therefore equality must hold and thus the path from the top ends at \(N = 1 + 2 + \dots +n\) with all the numbers \(\{1,2,\ldots ,n\}\) being close by. Let \(B\) be that position. + +![](data:image/jpeg;base64,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) + + +Consider the two left/right neighbors \(X\) and \(Y\) of the endpoint \(B\) . Assume that \(B\) is to the right of the midpoint of the bottom side, and complete the equilateral triangle + + + +as shown to an apex \(C\) . Consider the lightning strike from \(C\) hitting the bottom at \(D\) . It travels at least \(\lfloor n / 2 - 1 \rfloor\) steps, by construction. But the increases must be at least \(n + 1\) , \(n + 2\) , ...since \(1, 2, \ldots , n\) are close to the \(A \to B\) lightning path. Then the number at \(D\) is at least + +\[(n + 1) + (n + 2) + \dots +(n + (\lfloor n / 2 - 1\rfloor)) > 1 + 2 + \dots +n\] + +for \(n \geq 2018\) , contradiction. + + + +## \(\S 2\) Solutions to Day 2 + +## \(\S 2.1\) IMO 2018/4, proposed by Gurgen Asatryan (ARM) + +Available online at https://aops.com/community/p10632348. + +## Problem statement + +A site is any point \((x,y)\) in the plane for which \(x,y\in \{1,\ldots ,20\}\) . Initially, each of the 400 sites is unoccupied. Amy and Ben take turns placing stones on unoccupied sites, with Amy going first; Amy has the additional restriction that no two of her stones may be at a distance equal to \(\sqrt{5}\) . They stop once either player cannot move. Find the greatest \(K\) such that Amy can ensure that she places at least \(K\) stones. + +The answer is \(K = 100\) + +First, we show Amy can always place at least 100 stones. Indeed, treat the problem as a grid with checkerboard coloring. Then Amy can choose to always play on one of the 200 black squares. In this way, she can guarantee half the black squares, i.e. she can get \(\frac{1}{2}\cdot 200 = 100\) stones. + +Second, we show Ben can prevent Amy from placing more than 100 stones. Divide into several \(4\times 4\) squares and then further partition each \(4\times 4\) squares as shown in the grid below. + + + +The squares with each label form 4- cycles by knight jumps. For each such cycle, whenever Amy plays in the cycle, Ben plays in the opposite point of the cycle, preventing Amy from playing any more stones in that original cycle. Hence Amy can play at most in \(1 / 4\) of the stones, as desired. + + + +## \(\S 2.2\) IMO 2018/5, proposed by Bayarmagnai Gombodorj (MNG) + +Available online at https://aops.com/community/p10632353. + +## Problem statement + +Let \(a_{1}\) , \(a_{2}\) , ... be an infinite sequence of positive integers, and \(N\) a positive integer. Suppose that for all integers \(n \geq N\) , the expression + +\[\frac{a_{1}}{a_{2}} +\frac{a_{2}}{a_{3}} +\dots +\frac{a_{n - 1}}{a_{n}} +\frac{a_{n}}{a_{1}}\] + +is an integer. Prove that \((a_{n})\) is eventually constant. + +The condition implies that the difference + +\[S(n) = \frac{a_{n + 1} - a_{n}}{a_{1}} +\frac{a_{n}}{a_{n + 1}}\] + +is an integer for all \(n > N\) . We proceed by \(p\) - adic valuation only henceforth; fix a prime \(p\) . Then analyzing the \(\nu_{p}\) , we immediately get that for \(n > N\) : + +- If \(\nu_{p}(a_{n}) < \nu_{p}(a_{n + 1})\) , then \(\nu_{p}(a_{n + 1}) = \nu_{p}(a_{1})\) . + +- If \(\nu_{p}(a_{n}) = \nu_{p}(a_{n + 1})\) , no conclusion. + +- If \(\nu_{p}(a_{n}) > \nu_{p}(a_{n + 1})\) , then \(\nu_{p}(a_{n + 1}) \geq \nu_{p}(a_{1})\) . + +In other words: + +Claim — Let \(p\) be a prime. Consider the sequence \(\nu_{p}(a_{N + 1})\) , \(\nu_{p}(a_{N + 2})\) , .... Then either: + +- We have \(\nu_{p}(a_{N + 1}) \geq \nu_{p}(a_{N + 2}) \geq \ldots\) and so on, i.e. the sequence is weakly decreasing immediately; or + +- For some index \(K > N\) we have \(\nu_{p}(a_{K}) < \nu_{p}(a_{K + 1}) = \nu_{p}(a_{K + 2}) = \dots = \nu_{p}(a_{1})\) , i.e. the sequence "jumps" to \(\nu_{p}(a_{1})\) at some point and then stays there forever after. Note this requires \(\nu_{p}(a_{1}) > 0\) . + +A cartoon of the situation is drawn below. + +![](data:image/jpeg;base64,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) + + + + +As only finitely many primes \(p\) divide \(a_{1}\) , after some time \(\nu_{p}(a_{n})\) is fixed for all such \(p \mid a_{1}\) . Afterwards, the sequence satisfies \(a_{n + 1} \mid a_{n}\) for each \(n\) , and thus must be eventually constant. + +Remark. This solution is almost completely \(p\) - adic, in the sense that I think a similar result holds if one replaces \(a_{n} \in \mathbb{Z}\) by \(a_{n} \in \mathbb{Z}_{p}\) for any particular prime \(p\) . In other words, the primes almost do not talk to each other. + +There is one caveat: if \(x_{n}\) is an integer sequence such that \(\nu_{p}(x_{n})\) is eventually constant for each prime then \(x_{n}\) may not be constant. For example, take \(x_{n}\) to be the \(n\) th prime! That's why in the first claim (applied to co- finitely many of the primes), we need the stronger non- decreasing condition, rather than just eventually constant. + +Remark. An alternative approach is to show that, when the fractions \(a_{n} / a_{1}\) is written in simplest form for \(n = N + 1, N + 2, \ldots\) , the numerator and denominator are both weakly decreasing. Hence it must eventually be constant; in which case it equals \(\frac{1}{1}\) . + + + +## \(\S 2.3\) IMO 2018/6, proposed by Tomasz Ciesla (POL) + +Available online at https://aops.com/community/p10632360. + +## Problem statement + +A convex quadrilateral \(ABCD\) satisfies \(AB\cdot CD = BC\cdot DA\) . Point \(X\) lies inside \(ABCD\) so that + +\[\angle XAB = \angle XCD\quad \mathrm{and}\quad \angle XBC = \angle XDA.\] + +Prove that \(\angle BXA + \angle DXC = 180^{\circ}\) . + +We present two solutions by inversion. The first is the official one. The second is a solution via inversion, completed by USA5 Michael Ren. + +Official solution by inversion. In what follows a convex quadrilateral is called quasi- harmonic if \(AB\cdot CD = BC\cdot DA\) . + +Claim — A quasi- harmonic quadrilateral is determined up to similarity by its angles. + +Proof. Do some inequalities. + +Remark. This could be expected by degrees of freedom; a quadrilateral has four degrees of freedom up to similarity; the pseudo- harmonic condition is one while the measures of angles \(\angle A\) , \(\angle B\) , \(\angle C\) , \(\angle D\) (summing to \(360^{\circ}\) ) provide three degrees of freedom. (Note that the point \(X\) plays no role in this comment.) + +Performing an inversion at \(X\) , one obtains a second quasi- harmonic quadrilateral \(A^{*}B^{*}C^{*}D^{*}\) which has the same angles as the original one, \(\angle D^{*} = \angle A\) , \(\angle A^{*} = \angle B\) , and so on. Thus by the claim we obtain similarity + +\[D^{*}A^{*}B^{*}C^{*}\sim ABCD.\] + +If one then maps \(D^{*}A^{*}B^{*}C^{*}\) , onto \(ABCD\) , the image of \(X^{*}\) becomes a point isogonally conjugate to \(X\) . In other words, \(X\) has an isogonal conjugate in \(ABCD\) . + +It is well- known that this is equivalent to \(\angle BXA + \angle DXC = 180^{\circ}\) , for example by inscribing an ellipse with foci \(X\) and \(X^{*}\) . + +Second solution: "rhombus inversion", by Michael Ren. Since + +\[\frac{AB}{AD} = \frac{CB}{CD}\] + +and + +\[\frac{BA}{BC} = \frac{DA}{DC}\] + +it follows that \(B\) and \(D\) lie on an Apollonian circle \(\omega_{AC}\) through \(A\) and \(C\) , while \(A\) and \(C\) lie on an Apollonian circle \(\omega_{BD}\) through \(B\) and \(D\) . We let these two circles intersect at a point \(P\) inside \(ABCD\) . + +The main idea is then to perform an inversion about \(P\) with radius 1. We obtain: + + + +## Lemma + +The image of \(ABCD\) is a rhombus. + +Proof. By the inversion distance formula, we have + +\[\frac{1}{A^{\prime}B^{\prime}} = \frac{PA}{AB}\cdot PB = \frac{PC}{BC}\cdot PB = \frac{1}{B^{\prime}C^{\prime}}\] + +and so \(A^{\prime}B^{\prime} = B^{\prime}C^{\prime}\) . In a similar way, we derive \(B^{\prime}C^{\prime} = C^{\prime}D^{\prime} = D^{\prime}A^{\prime}\) , so the image is a rhombus as claimed. \(\square\) + +Let us now translate the angle conditions. We were given that \(\angle XAB = \angle XCD\) , but + +\[\angle XAB = \angle XAP + \angle PAB = \angle PX'A' + \angle A'B'P\] + +\[\angle XCD = \angle XCP + \angle PCD = \angle PX'C' + \angle C'D'P\] + +so subtracting these gives + +\[\begin{array}{r l} & {\angle A^{\prime}X^{\prime}C^{\prime} = \angle A^{\prime}B^{\prime}P + \angle P D^{\prime}C^{\prime} = \angle (A^{\prime}B^{\prime},B^{\prime}P) + \angle (P D^{\prime},C^{\prime}D^{\prime})}\\ & {\qquad = \angle (A^{\prime}B^{\prime},B^{\prime}P) + \angle (P D^{\prime},A^{\prime}B^{\prime}) = \angle D^{\prime}P B^{\prime}.} \end{array} \quad (1)\] + +since \(\overline{{A^{\prime}B^{\prime}}}\parallel \overline{{C^{\prime}D^{\prime}}}\) . Similarly, we obtain + +\[\angle B^{\prime}X^{\prime}D^{\prime} = \angle A^{\prime}PC^{\prime}. \quad (2)\] + +We now translate the desired condition. Since + +\[\angle A X B = \angle A X P + \angle P X B = \angle P A^{\prime}X^{\prime} + \angle X^{\prime}B^{\prime}P\] \[\angle C X D = \angle C X P + \angle P X D = \angle P C^{\prime}X^{\prime} + \angle X^{\prime}D P^{\prime}\] + +we compute + +\[\angle A X B + \angle C X D = (\angle P A^{\prime}X^{\prime} + \angle X^{\prime}B^{\prime}P) + (\angle P C^{\prime}X^{\prime} + \angle X^{\prime}D^{\prime}P)\] \[\qquad = - \left[(\angle A^{\prime}X^{\prime}P + \angle X^{\prime}P A^{\prime}) + (\angle P X^{\prime}B^{\prime} + \angle B^{\prime}P X^{\prime})\right]\] \[\qquad - \left[(\angle C^{\prime}X^{\prime}P + \angle X^{\prime}P C^{\prime}) + (\angle P X^{\prime}D^{\prime} + \angle D^{\prime}P X^{\prime})\right]\] \[\qquad = \left[\angle P X^{\prime}A^{\prime} + \angle B X^{\prime}P + \angle P X^{\prime}C^{\prime} + \angle D^{\prime}X^{\prime}P\right]\] \[\qquad +\left[\angle A^{\prime}P X^{\prime} + \angle X^{\prime}P B^{\prime} + \angle C^{\prime}P X^{\prime} + \angle X^{\prime}P D^{\prime}\right]\] \[\qquad = \angle A^{\prime}P B^{\prime} + \angle C^{\prime}P D^{\prime} + \angle B^{\prime}X^{\prime}C + \angle D^{\prime}X^{\prime}A\] + +and we wish to show this is equal to zero, i.e. the desired becomes + +\[\angle A^{\prime}P B^{\prime} + \angle C^{\prime}P D^{\prime} + \angle B^{\prime}X^{\prime}C + \angle D^{\prime}X^{\prime}A = 0. \quad (3)\] + +In other words, the problem is to show (1) and (2) implies (3). + +Henceforth drop apostrophes. Here is the inverted diagram (with apostrophes dropped). + + +![](data:image/jpeg;base64,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) + + +Let \(Q\) denote the reflection of \(P\) and let \(Y\) denote the second intersection of \((BQC)\) and \((AQD)\) . Then + +\[-\angle AXC = -\angle DPB = \angle BQD = \angle BQY + \angle YQD = \angle BCY + \angle YAD\] \[= \angle (BC,CY) + \angle (YA,AD) = \angle YCA = -\angle AYC.\] + +Hence \(XACY\) is concyclic; similarly \(XBDY\) is concyclic. + +Claim — \(X \neq Y\) . + +Proof. To see this: Work pre- inversion assuming \(AB < AC\) . Then \(Q\) was the center of \(\omega_{BD}\) . If \(T\) was the second intersection of \(BA\) with \((QBC)\) , then \(QB = QD = QT = \sqrt{QA \cdot QC}\) , by shooting lemma. Since \(\angle BAD < 180^\circ\) , it follows \((QBCY)\) encloses \(ABCD\) (pre- inversion). (This part is where the hypothesis that \(ABCD\) is convex with \(X\) inside is used.) \(\square\) + +Finally, we do an angle chase to finish: + +\[\begin{array}{r l} & {\angle D X A = \angle D X Y + \angle Y X A = \angle D B Y + \angle Y C A}\\ & {\qquad = \angle (D B,Y B) + \angle (C Y,C A) = \angle C Y B + 90^{\circ}}\\ & {\qquad = \angle C Q B + 90^{\circ} = -\angle A P B + 90^{\circ}.} \end{array} \quad (4)\] + +Similarly, + +\[\angle BXC = \angle DPC + 90^\circ . \quad (5)\] + +Summing (4) and (5) gives (3). + +Remark. A difficult part of the problem in many solutions is that the conclusion is false in the directed sense, if the point \(X\) is allowed to lie outside the quadrilateral. We are saved in the first solution because the equivalence of the isogonal conjugation requires \(X\) inside the quadrilateral. On the other hand, in the second solution, the issue appears in the presence + + + +of the second point \(Y\) . + +